Jump to content

Can you move in space?


Recommended Posts

45 minutes ago, Strange said:

I don't think you can generate an isolated pulse of magnetism that flies out the back of the craft to impart momentum. Because, as far as I know, there is no such thing as a "pulse of magnetic field" as a freely moving entity that can carry momentum. (And no one has told me I am mistaken yet - and that usually happens pretty quickly when I am wrong!)

A quick google suggests that it is possible

copied from google :

‘Pulsed field magnets are used in research in fields such as materials science to study the effect of strong magnetic fields, since they canproduce stronger fields than continuous magnets. ... The magnetic fieldproduced by pulsed field magnets canreach between 50 and 100 T, and lasts several tens of milliseconds.’

im guessing that we are talking about a theory that hasn’t come up before which is why people aren’t rushing in to correct anyone and on that I do vaguely recall reading here that most theories have been put forward before - I’m guessing this type of theory doesn’t come up too often. 

Even if photons with momentum are produced as a result of producing magnetic field pulses between the wires as described, I can’t see the amount of momentum they have could produce the amount of propulsion that a magnetic field could. Case in point of pulsing magnetic fields having the ability to create massive amounts of moment are gauss weapons being developed to accelerate projectiles. I’m not suggesting that my proposed system is an adaptation of a glass weapon to generate propulsion. I’m just using the reference to gauss weapons to demonstrate how magnetic fields can produce momentum.

 

 

Edited by MPMin
Link to comment
Share on other sites

34 minutes ago, MPMin said:

A quick google suggests that it is possible

I think that is rather different. By passing an intermittent current through a wire, you will get an intermittent (pulsing) magnetic field around that wire.

What you won't get (I don't think) is a little puff of magnetic field flying off by itself. 

What you will get (I am sure) is electromagnetic radiation flying away because of the pulsed magnetic field.

37 minutes ago, MPMin said:

Even if photons with momentum are produced as a result of producing magnetic field pulses between the wires as described, I can’t see the amount of momentum they have could produce the amount of propulsion that a magnetic field could.

Why. Can you quantify this?

Using electromagnetic pulses does not generate much thrust though. This is because of E=mc2. The momentum associated with a small amount of solid propellant is huge compared to the amount of energy in an electromagnetic wave.

A magnetic field can be used, very effectively, to produce propulsion if you have, say, a static magnet in a train and a changing magnetic field fixed to the rails (or vice versa). And that is pretty efficient (it is basically an electric motor). But that obviously doesn't work in space.

Link to comment
Share on other sites

1 hour ago, MPMin said:

I’m not suggesting that my proposed system is an adaptation of a glass weapon to generate propulsion.

True. Since you insist on analogies the correct analogy is that your system is a Gauss gun with the projectile welded solid in the barrel, trying to propel iself. (In the right context quite funny I think.)

I think @Strange is correct, per Maxwell an EMP will contain a mix of energies. But one may be dominant. There is a short text on https://en.m.wikipedia.org/wiki/Electromagnetic_pulse

 

Quote

EMP energy may be transferred in any of four forms:

Electric field

Magnetic field

Electromagnetic radiation

Electrical conduction

Due to Maxwell's equations, a pulse of any one form of electromagnetic energy will always be accompanied by the other forms, however in a typical pulse one form will dominate.

In general, only radiation acts over long distances, with the others acting over short distances. There are a few exceptions, such as a solar magnetic flare.

 

Edited by Ghideon
Grammar
Link to comment
Share on other sites

7 hours ago, Ghideon said:

True. Since you insist on analogies the correct analogy is that your system is a Gauss gun with the projectile welded solid in the barrel, trying to propel iself. (In the right context quite funny I think.)

This is an inaccurate representation of my idea and still makes me wonder if I’ve explained myself adequately. You are proposing that the magnetic field is trying to push itself off of its mounting where as I’m saying its generating a ‘casually detached’ magnetic field to push or pull on.

i understand that generating a magnetic pulse will also produce other forms of energy which I understand to mean that 100% of the energy used to produce the magnetic field will not be 100% efficient, i never suggested it would, Ive said all along the system will require an energy input. If this is used in space perhaps the low temperature will cause the wire to have no resistance but that’s the best case scenario as some of the energy will be lost in other forms.

Its still not clear to me if the law of conservation of energy applies as energy can be lost from the system and on that, energy can be added to the system as well.

At this stage the defining question is:

D6842A6C-3ACB-4FB4-AE89-7A8B6850C3C9.thumb.jpeg.887c4b334680428f6c6ee41ea70d7c9b.jpeg

will the situation above produce a physical force on the wire as shown by the red arrow and if not, why not in terms of the magnetic field interactions?

Edited by MPMin
Link to comment
Share on other sites

7 hours ago, MPMin said:

where as I’m saying its generating a ‘casually detached’ magnetic field to push or pull on

You cannot generate such a field. If you could, you could not push against it.

8 hours ago, MPMin said:

i understand that generating a magnetic pulse will also produce other forms of energy which I understand to mean that 100% of the energy used to produce the magnetic field will not be 100% efficient, i never suggested it would, Ive said all along the system will require an energy input. If this is used in space perhaps the low temperature will cause the wire to have no resistance but that’s the best case scenario as some of the energy will be lost in other forms.

No, you haven't understood at all.

You cannot generate a magnetic pulse that will drive your craft. A magnetic field (pulsed or otherwise) only exists around the wire creating it. (And with distance, the field will rapidly drop to zero.)

What you can generate is electromagnetic radiation that can be radiated from the craft. Radiate this directionally and you will generate thrust.

8 hours ago, MPMin said:

Its still not clear to me if the law of conservation of energy applies as energy can be lost from the system and on that, energy can be added to the system as well.

Conservation of energy applies as long as you take into account the inputs and outputs from the system.

8 hours ago, MPMin said:

will the situation above produce a physical force on the wire as shown by the red arrow and if not, why not in terms of the magnetic field interactions?

Of course it will (Faraday's laws). But it is irrelevant. 

Isn't this where we came in? You don't seem to have changed your position at all.

Link to comment
Share on other sites

Its hard for me to accept what you are saying when what you are saying is unclear to me.

You say I haven’t understood but the only thing I’m having trouble understanding is how you are drawing your conclusions. When you make statements that are out of context i have trouble understanding your reasoning. I know what you all have been saying but you don’t seem to understand what I’ve been saying so I’m struggling to accept your reasoning on this matter. A lot of what you are saying is really out of context, for example, you say with distance the field will drop to zero, in general terms yes but it really depends how small a percentage of the original field strength you want to regard as negligible (rather than actual ‘zero’ ) and i am well aware that field strength is inversely proportional to the distance squared but again this comment is out of context as I’m not talking about huge distances between the wires in my proposal. You also say that such a field can only exist around the wire creating it as if to suggest my proposal wont produce a magnetic field but if you look at my drawing that’s exactly what I’ve drawn - a magnetic field around the wire, so when i read all these out of context comments, how can you expect me to just accept what you are saying. Further more you then you go on to say that of course that magnetic field will produce a force on the wire in the diagram (after you more or less said it wouldn’t produce a magnetic field) on the wire. We’ve gone through four or five pages of you telling me i cant violate newtons laws ( Id like to add a note here to Ghideon while you are laughing at me with my finger in my guass gun I’m laughing at you coming across as being religious defending your god newton and his commandments I don’t mean that in a bad way I did see the funny side of what you said) when as it turns out now you add a disclaimer about the conservation of energy saying they don’t apply but oh they do if you measure all the immeasurables. I do appreciate all the time you’ve taken on this with me and I’d even more appreciate it if you all were a little more objective about this and really try understand what I’m saying first before just jumping to conclusions. I’m sorry i frightened you all with suggesting its a reaction less drive maybe it’s not reactionless, does that make you all feel better? Ive come to you all as the experts in your field and I’m looking for certain answers, not suppositions. If I’m wrong then I’m wrong that’s fine but please show me in no uncertain terms that I’m wrong. 

Link to comment
Share on other sites

32 minutes ago, MPMin said:

You say I haven’t understood but the only thing I’m having trouble understanding is how you are drawing your conclusions.

From a basic knowledge of physics.

32 minutes ago, MPMin said:

 I know what you all have been saying but you don’t seem to understand what I’ve been saying so I’m struggling to accept your reasoning on this matter.

The trouble is that you are not able to explain what you think will happen in terms of physics (if you were able to, you would also realise it couldn't happen).

You seem to want to caused a pulse of "magnetic energy" to be created by kind of bouncing a pulse from one wire off another wire (using the magnetic field generated in that second wire). 

Is that roughly it?

Given that your idea is not very clear, all I can do is try and explain what is possible. 

35 minutes ago, MPMin said:

A lot of what you are saying is really out of context, for example, you say with distance the field will drop to zero, in general terms yes but it really depends how small a percentage of the original field strength you want to regard as negligible (rather than actual ‘zero’ ) and i am well aware that field strength is inversely proportional to the distance squared but again this comment is out of context as I’m not talking about huge distances between the wires in my proposal. 

In the case of a magnetic field, it falls off with a cube law, I think.

But that is not really the point. The magnetic field is associated with the wire that generates it. Even if you generate a directional magnetic field (as in a bar magnet, for example, which you can do with a col) it won't provide any thrust (unless there is some magnetic material for it to push against).

If you pulse your magnetic field, all that happens is that it goes on and off. It doesn't radiate or travel away from your craft.

38 minutes ago, MPMin said:

You also say that such a field can only exist around the wire creating it as if to suggest my proposal wont produce a magnetic field but if you look at my drawing that’s exactly what I’ve drawn - a magnetic field around the wire

But if it is around the wire it cannot provide any thrust to the craft. You need some "thing" (with energy and momentum) to leave the craft to provide thrust. A magnetic field cannot be that thing.

40 minutes ago, MPMin said:

Further more you then you go on to say that of course that magnetic field will produce a force on the wire in the diagram (after you more or less said it wouldn’t produce a magnetic field) on the wire.

I never said it would not generate a magnetic field. Of course it will if it is carrying a current. What I said was that you can't create a field that is detached from the wires and flies out the back of the craft to provide thrust.

41 minutes ago, MPMin said:

when as it turns out now you add a disclaimer about the conservation of energy saying they don’t apply

I never said that.

41 minutes ago, MPMin said:

I’m sorry i frightened you all with suggesting its a reaction less drive

You didn't frighten anybody. We have seen all these arguments many times before. Occasionally people are able to see where they have gone wrong...

42 minutes ago, MPMin said:

Ive come to you all as the experts in your field and I’m looking for certain answers, not suppositions. If I’m wrong then I’m wrong that’s fine but please show me in no uncertain terms that I’m wrong. 

I don't know how else to explain.

1. You cannot fire pulses of magnetic field out of the back of you craft to provide thrust (magnetic fields just don't work like that).

2. You can't generate a force on the craft from the forces on the wires internal to the craft. If one of the wires is external and one is internal, then you can potentially build a linear motor using the force between them.

3. You can generate electromagnetic radiation and use that to generate thrust.

http://hyperphysics.phy-astr.gsu.edu/hbase/magnetic/magfie.html#c1

Link to comment
Share on other sites

2 hours ago, MPMin said:

Its hard for me to accept what you are saying when what you are saying is unclear to me

Maybe you can provide some references to material you base your claims upon? Material you have read and understod. If we base the discussion on some sufficiently detailed sources any misunderstandings may be sorted out?

Link to comment
Share on other sites

2 hours ago, MPMin said:

 We’ve gone through four or five pages of you telling me i cant violate newtons laws ( Id like to add a note here to Ghideon while you are laughing at me with my finger in my guass gun I’m laughing at you coming across as being religious defending your god newton and his commandments

...

If I’m wrong then I’m wrong that’s fine but please show me in no uncertain terms that I’m wrong. 

These two sentiments are in direct conflict. Citing Newton's laws is showing you in no uncertain terms you are wrong. Pointing to your own ignorance of physics isn't an effective counterargument.

Link to comment
Share on other sites

17 hours ago, Strange said:

2. You can't generate a force on the craft from the forces on the wires internal to the craft. If one of the wires is external and one is internal, then you can potentially build a linear motor using the force between them

Someone here said that if any of the wires are attached to the craft then its considered internal, are you using ‘internal’ in the same context?

 

15 hours ago, Ghideon said:

Maybe you can provide some references to material you base your claims upon? Material you have read and understod. If we base the discussion on some sufficiently detailed sources any misunderstandings may be sorted out?

Perhaps if i explain my theory in seperate aspects you could tell me which part of my theory is wrong? Is it ok if i reconstruct my theory in a ‘if then’ sequence?

 

15 hours ago, swansont said:

These two sentiments are in direct conflict. Citing Newton's laws is showing you in no uncertain terms you are wrong. Pointing to your own ignorance of physics isn't an effective counterargument.

If the system requires an energy input and loses energy from the system, can it be said in no uncertain terms that it’s violating newtons law of conservation?

Link to comment
Share on other sites

44 minutes ago, MPMin said:

Perhaps if i explain my theory in seperate aspects you could tell me which part of my theory is wrong? Is it ok if i reconstruct my theory in a ‘if then’ sequence?

A theory should be based on something, papers or books from mainstream science. What postulates do you start from? Can you provide references to material we can start from? Something you have read and understod and based your work on?

Link to comment
Share on other sites

2 hours ago, MPMin said:

Someone here said that if any of the wires are attached to the craft then its considered internal, are you using ‘internal’ in the same context?

If all of the wires are fixed to the craft then it is “internal” (in the sense of, part of the system).

 

2 hours ago, MPMin said:

Perhaps if i explain my theory in seperate aspects you could tell me which part of my theory is wrong? Is it ok if i reconstruct my theory in a ‘if then’ sequence?

Sounds like a good idea

3 hours ago, MPMin said:

If the system requires an energy input and loses energy from the system, can it be said in no uncertain terms that it’s violating newtons law of conservation?

The conservation laws (which are not due to Newton) and Newton's laws are never violated.

If you think you have found a case where they are,then you have missed something.

Link to comment
Share on other sites

3 hours ago, MPMin said:

If the system requires an energy input and loses energy from the system, can it be said in no uncertain terms that it’s violating newtons law of conservation?

Conservation of momentum?

No. Energy and momentum are not the same thing; the behavior of a system will depend on the details of the energy input and output. You can have a situation where you exert no net force on the system, in which case momentum is conserved. You can also have a net force, in which case momentum would not be conserved. Note, however, that not conserving momentum in such a case is not a violation of the law — it's the expected result.

Momentum conservation does not get violated.

 

Link to comment
Share on other sites

1. For my theory to work, firstly relies on the fact that two wires in parallel, in reasonably close proximity to each other, carrying a continuous current will exert a force on each other.

https://www.khanacademy.org/science/physics/magnetic-forces-and-magnetic-fields/magnetic-field-current-carrying-wire/v/magnetism-7

 

2. The next principle my theory relies upon is that magnetic fields travel and it needs to be able to travel to become an emp. 

https://physics.stackexchange.com/questions/299947/does-electric-and-magnetic-fields-travel-or-they-just-appear-in-space

https://en.m.wikipedia.org/wiki/Electromagnetic_pulse

 

3. The next principle relies on an emp being able push or pull  a wire already carrying a current. Unfortunately i cant find any references exploring this thus I’m assuming based on the above; that if a continuous magnetic field (created by a continuous current) will exert a force on a wire then an intermittent magnetic field or emp will  as well?

 

 

Link to comment
Share on other sites

13 minutes ago, MPMin said:

1. For my theory to work, firstly relies on the fact that two wires in parallel, in reasonably close proximity to each other, carrying a continuous current will exert a force on each other.

https://www.khanacademy.org/science/physics/magnetic-forces-and-magnetic-fields/magnetic-field-current-carrying-wire/v/magnetism-7

OK.

13 minutes ago, MPMin said:

2. The next principle my theory relies upon is that magnetic fields travel and it needs to be able to travel to become an emp. 

https://physics.stackexchange.com/questions/299947/does-electric-and-magnetic-fields-travel-or-they-just-appear-in-space

https://en.m.wikipedia.org/wiki/Electromagnetic_pulse

Yes, the magnetic field spreads out from its source at c.

It doesn't need to "travel" to become an EMP, you just need a changing magnetic (or electric) field. They will travel at c, by definition.

Quote

3. The next principle relies on an emp being able push or pull  a wire already carrying a current. Unfortunately i cant find any references exploring this thus I’m assuming based on the above; that if a continuous magnetic field (created by a continuous current) will exert a force on a wire then an intermittent magnetic field or emp will  as well?

Yes, a static current will produce a static field, which will create a static force on another wire carrying current. 

And yes, if either current is changes, then the field will change resulting in a changing force. So you can get a pulsing force between the two wires.

 

 

Link to comment
Share on other sites

8 hours ago, MPMin said:

Do you agree with this animation then? please forgive my drawing ability

Off the top of my head (it is getting late), I think the force will be at right angles to that; in other words, either up or down, in your diagram (depending on the direction of the currents).

I'm sire someone who s feeling sharper will confirm or deny !

Having woken up, I realised this was wrong! Your diagram is correct (assuming the currents are in opposite directions).

Link to comment
Share on other sites

Currents are as shown in the wires.

the animation above depicts one basic cycle, at the end of the cycle the process repeats to generate more force on the wire. This process is happening at an extremely fast rate enabling many pulses of force to occur per second so that even if the force per cycle was very small, the cumulative effect could be a large force.

My theory relies on the fact that the magnetic pulse will only cause a force on the wire as shown. The wire experiencing the force, while being attached to the craft will push the craft and the other wire will be unaffected as its not carrying a current in that moment.

 

 

Link to comment
Share on other sites

7 minutes ago, MPMin said:

My theory relies on the fact that the magnetic pulse will only cause a force on the wire as shown. The wire experiencing the force, while being attached to the craft will push the craft and the other wire will be unaffected as its not carrying a current in that moment.

Aha.

I think I see what you are thinking now. You are suggesting that the magnetic field will exert a force on the second wire, with no reaction on the source (the first wire) thus "getting around" Newton's third law.

I would need to think about why this won't work. (But I can guarantee it won't.)

Maybe with this clearer understanding of what you are saying, someone else will come up with a good explanation.

Link to comment
Share on other sites

19 minutes ago, Strange said:

I think I see what you are thinking now. You are suggesting that the magnetic field will exert a force on the second wire, with no reaction on the source (the first wire) thus "getting around" Newton's third law.

Yes, that is what i am proposing.

Just in case it does work, there is also the possibility of utilising the second magnetic pulse as it arrives back at the source wire, the source wire could then have the current pulsed in the other direction so that it is attracted to the on coming magnetic pulse thus producing a force at the originals source wire in the same direction. However, There’s no point exploring this if the above cycle wont work.

Link to comment
Share on other sites

Disclaimer: I’ve access to phone and paper only, not able to produce good pictures. There is no analysis of how possible it is to create the device.

Here is the initial situation at time t=0. A and B are mounted in a rig, cables can't be moved independently. Cables are straight and seen running straight into the paper. Current is switched on in cable A.The rig is big so time of travel at speed of light across the rig is not neglectible.

C2EE2EDD-17E3-4BF0-8E6D-6E953B9B874D.thumb.jpeg.3c5f5101a53ff90c7791e3c3b343a6eb.jpeg

At some small time t1some amount of electromagnetic radiation have radiated away and the current is switched of. There will be magnetic and electric energy. We chose to model the radiation using particles. There are probably other usable models. Important thing is that cables A and B are not attracted by force (Ampere’s force law) but act more like antennas far apart.

 9C693295-B19E-4421-8234-1FA6D1320052.thumb.jpeg.be1c043153ddc980a43c2539ac54f1bb.jpeg

Here is a crucial simplification that other readers may have opinions about: the radiated pulse is symmetrical and we do not care about the exact composition; photons, electrons or other. We only care about the momentum carried by the radiated particles. Each individual particle carries momentum directed away from the center of the cable A.

There are two components of the momentum, x and y. Since momentum in y direction is never part of an interaction and has total sum of zero we simplify and analyze x-component only. We will analyze the particles that will later interact with a field around cable B and the particles radiated in 180 degree opposite direction. This simplification allows for a new diagram showing two pulses traveling in opposite directions carrying momentum P and -P. Note the convention, I prefer the pulse later reaching cable B to have positive sign of momentum P.

967A709F-C541-435A-A4CD-8F2F24443D49.thumb.jpeg.4a9a674e7dc8ecdbe37d928c4cea3576.jpeg

At time t=t2 the pulse reaches cable B. Again a simplification. The cable is surrounded by a symmetrical field interacting with incoming pulse. then, the maximum amount of momentum gained by interaction with the pulse is P. We do not need to model the exact interaction of the fields. There is no magical way to gain more momentum than what the incoming pulse carries.

1F6B2327-25C3-4BBA-B67B-EEAE01E70192.thumb.jpeg.fa80304b8f7fc723fc398ed15426f097.jpeg

The result is that the complete rig with both cables starts moving to the left. 

Now some final notes. Note that I discuss only the x components of the circular pulse. The movement of the rig is not due to momentum P of left pulse. It it is momentum -P of right pulse that does the job. -P is just prevented from doing so until cable B stops the left pulse momentum P. If cable A had sent only the burst -P the movement would have begun at t=0. Now the rig is stationary until time t=t2. 

Had cable A sent only pulse P then the rig would have moved to the right for a short while and then stopped when cable B interact with left pulse. Left pulse is not adding momentum P, left pulse prevents the rig from moving until cable B stops the left pulse. 

 

 

Edited by Ghideon
Format, grammar. No fun doing this on a phone.
Link to comment
Share on other sites

Guest
This topic is now closed to further replies.
×
×
  • Create New...

Important Information

We have placed cookies on your device to help make this website better. You can adjust your cookie settings, otherwise we'll assume you're okay to continue.